57
Unlimited permission is granted free of charge to print or photocopy all pages of this work for educational, not-for-profit use by faculty members, students or board candidates. DR. FAHMI KHAN: www.fakhanqal.com Clinical Quiz series for medical students and board candidates Paper 2: Contains 120 best of five questions; 60 MCQs (False & true); 20 slides and answer key Dr.fahmi khan

DR. FAHMI KHAN:  · 2017-10-18 · 1. A 71-year-old man undergoes a partial colectomy for colon cancer. He develops temperature of 38.6º C on the third post operative day and rigors

  • Upload
    others

  • View
    1

  • Download
    0

Embed Size (px)

Citation preview

Page 1: DR. FAHMI KHAN:  · 2017-10-18 · 1. A 71-year-old man undergoes a partial colectomy for colon cancer. He develops temperature of 38.6º C on the third post operative day and rigors

Unlimited permission is granted free of charge to print or photocopy all pages of this work for educational, not-for-profit use by faculty members, students or board candidates.

DR. FAHMI KHAN: www.fakhanqal.com

Clinical Quiz series for medical students and board candidates Paper 2: Contains 120 best of five questions; 60 MCQs (False & true); 20 slides and answer key

Dr.fahmi khan

Page 2: DR. FAHMI KHAN:  · 2017-10-18 · 1. A 71-year-old man undergoes a partial colectomy for colon cancer. He develops temperature of 38.6º C on the third post operative day and rigors

2

A: For each question below choose the single best a nswer

1. A 71-year-old man undergoes a partial colectomy for colon cancer. He develops temperature of 38.6º C on the third post operative day and rigors. On examination, his Bp is 100/60 mmHg, HR 100 and RR 24 and there is rales at the base of his right lung. The wound is clean and the remainder of the examination is normal. Investigations: chest X-ray shows right lower lobe pneumonia, Hemoglobin 12.9 g/dl, platelets 300 X 109/l, WBC 15.3 X 109/l, ESR 81 mm in the first hour, U&Es and LFTs are normal. Blood and sputum are sent to laboratory for culture and Gram stain. Vancomycin and piperacillin/tazobactam are initiated empirically. The following two days he remains febrile and dyspneic. Blood and sputum cultures reveal Klebsiella pneumoniae sensitive to cefepime, piperacillin/tazobactam, imipenem, gentamycin and tobramycin but resistant to ciprofloxacin, cefazolin and ceftazidime. Which of the following is the most appropriate mana gement option at this time?

A) Continue same antibiotic B) Add gentamycin C) Discontinue vancomycin and add gentamycin D) Discontinue piperacillin/tazobactam and add cefepime E) Discontinue current antibiotics and start imipenem

2. A 60-year-old man with end stage renal failure from chronic glomerunephritis presents with acute onset of gross hematuria and mild flank pain. He has been on hemodialysis for 4 years, and his course otherwise been uneventful. He was afebrile and the hematuria resolved without intervention. Which of the following is the most appropriate acti on at this time?

A) Angiography B) Computed tomography (to rule out acquired cystic kidney disease) C) Renal ultrasound D) Intravenous pyelography E) None of the above

3. A 65-year-old woman comes for a preoperative evaluation before elective right knee arthroplasty. She had acute myocardial infarction 4 months ago with paclitaxel drug-eluting stent in left anterior descending (LAD) coronary artery. She has hypertension and DM. Current medications are clopidogril, metoprolol, atorvastatin, losartan, metformin, aspirin, acetaminophen and tramadol. On examination Vital signs are normal. Other than evidence of bony hyper atrophy of the knees and small effusion in the right knee, the remainder of the exam is unremarkable. ECG shows normal sinus rhythm; Q waves in inferior leads and LVH. Which of the following is the best preoperative man agement?

A) Postpone surgery for 2 months. B) Postpone surgery for 8 months. C) proceed with surgery; continue aspirin but temporarily stop clopidogril D) proceed with surgery; continue both aspirin and clopidogril E) proceed with surgery; temporarily discontinue aspirin and clopidogril

4. Pseudopseudohypoparathyroidism is characterized by all of the following EXCEPT:

A) Hypocalcemia B) G5 mutation C) Albright’ s hereditary osteodystrophy D) inheritance of the genetic defect from the father E) Normal urinary c-AMP response to parathyroid hormone

Page 3: DR. FAHMI KHAN:  · 2017-10-18 · 1. A 71-year-old man undergoes a partial colectomy for colon cancer. He develops temperature of 38.6º C on the third post operative day and rigors

3

5. An 82-year-old woman with a history of Hashimoto's thyroiditis is evaluated for a rapidly expanding thyroid mass and progressive dysphagia and dyspnea. She has lost 2.2 kg over the preceding 6 weeks. On examination: BP118/78 mm Hg, PR 86/min. Thyroid examination reveals a firm 5-cm left thyroid mass that moves poorly with swallowing. Respiratory stridor is evident. Fine needle aspiration of the mass shows numerous uniform lymphocytes. CT scan of the neck shows the thyroid to completely encircle the trachea. Which of the following is the most likely diagnosis ?

A) Medullary thyroid cancer B) Anaplastic thyroid cancer C) Thyroid hemorrhage D) Thyroid lymphoma E) Laryngeal cancer

6. A 25-year-old female, with systemic lupus erythematosus (SLE) known to have Anti-Ro/SSA antibodies in her blood is in remission; current treatment consists of azathioprine 75 mg/d and prednisone 5 mg/d. Last year she had a life-threatening exacerbation of her disease. She now strongly desires to become pregnant. Which of the following is the least appropriate act ion to take?

A) Warn her that exacerbations can occur in the first trimester and the postpartum period. B) Advise her that risk of spontaneous abortion is high. C) Advise her that fetal loss rates are higher if anticardiolipin antibodies are detected in her

serum. D) Stop prednisone just before she attempts to become pregnant. E) Tell her that neonatal lupus syndrome can occur.

7. A 24-year-old female known to have bronchial asthma two years ago, presents to your clinic for follow-up. She is currently using a salbutamol inhaler 100mcg prn combined with beclomethasone dipropionate inhaler 200mcg bd. Despite this her asthma is not well controlled. On examination her chest is clear and she has a good inhaler technique. What is the most appropriate next step in managemen t?

A) Increase beclomethasone dipropionate to 400mcg bid B) Switch steroid to fluticasone propionate C) Trial of leukotriene receptor antagonist D) Add salmeterol E) Add tiotropium

8. A 27-years-old woman known to have hyperprolactinemia due to pituitary adenoma (MRI diagnosis). She was given bromocriptine, which normalized her menstruation as well as her prolactin level. She informed her family physician that she was pregnant and was not sure whether she should stop taking her bromocriptine. What is the most appropriate next step in managemen t?

A) Discontinue bromocriptine and remind patients to contact their physicians if they have new-onset headaches or vision changes with follow-up every three months.

B) Discontinue bromocriptine and start cabergoline with measuring prolactin and visual field perimetry every 2 months during pregnancy.

C) Urgent pituitary imaging (preferably magnetic resonance imaging). D) Neurosurgeon referral. E) No intervention recommended.

Page 4: DR. FAHMI KHAN:  · 2017-10-18 · 1. A 71-year-old man undergoes a partial colectomy for colon cancer. He develops temperature of 38.6º C on the third post operative day and rigors

4

9. A 37-year-old woman presents for review. She is 26 weeks pregnant and has had no problems with her pregnancy to date. Blood pressure is 144/92 mmHg, a rise from her booking reading of 110/80 mmHg. Urine dipstick is negative for protein, leukocyte and blood. What is the most appropriate description of her con dition?

A) Moderate pre-eclampsia. B) Mild pre-eclampsia. C) Gestational hypertension. D) Normal physiological change in blood pressure. E) Pre-existing hypertension.

10. A 64-year-old man presents with a eight-month history of generalized weakness. On examination he has fasciculation and weakness in both arms with absent reflexes. Examination of the lower limbs reveal increased tone and exaggerated reflexes. Sensation was normal and there were no cerebellar signs. What is the most likely diagnosis?

A) Lead poisoning. B) Motor neuron disease. C) Vitamin B12 deficiency. D) Syringomyelia. E) Multiple sclerosis.

11. A new test to screen for pulmonary embolism (PE) is used in 100 patients who present to the Emergency Department. The test is positive in 30 of the 40 patients who are proven to have a PE. Of the remaining 60 patients, only 5 have a positive test. What is the sensitivity of the new test?

A) 8.25% B) 25% C) 40% D) 66.66% E) 75%

12. A 22-year-old woman has a congenital anemia. She has required multiple transfusions of red blood cells for many years. She now has no significant findings on physical examination. Laboratory studies now show a serum AST of 74 U/L and ALT 75 U/L with albumin 3.6 g/dL. Which of the following microscopic findings would m ost likely appear in a liver biopsy?

A) Steatosis in hepatocytes B) Bilirubin in canaliculi C) Hemosiderin in hepatocytes D) Glycogen in hepatocytes E) Amyloid in portal triads

13. You are caring for a patient on mechanical ventilation in the intensive care unit. Whenever the patient initiates a breath, no matter her spontaneous respiratory rate, she gets a fixed volume breath from the machine that does not change from breath to breath. After receiving a dose of sedation, she does not initiate any breaths, but the machine delivers the same volume breath at periodic fixed intervals during this time. Which of the following modes of mechanical ventilation is this patient receiving?

A) Assist control B) Continuous positive airway pressure C) Pressure control D) Pressure support E) Synchronized intermittent mandatory ventilation (SIMV)

Page 5: DR. FAHMI KHAN:  · 2017-10-18 · 1. A 71-year-old man undergoes a partial colectomy for colon cancer. He develops temperature of 38.6º C on the third post operative day and rigors

5

14. A 74-year-old woman who has degenerative joint disease and is scheduled to undergo a total knee replacement under spinal anesthesia. She has hypertension treated with a beta-adrenergic blocking agent. She had an appendectomy and a total abdominal hysterectomy. She has not had chest pain or dyspnea. Pulse rate is 60 per minute, and respirations are 14 per minute. Blood pressure is 140/92 mm Hg. Physical examination is normal except for a small left knee effusion, decreased left knee flexion and crepitus on left knee movement. A chest radiograph and electrocardiogram are normal, and hemoglobin is 14.0 g/dL. Which of the following should you recommend preoper atively?

A) Temporary discontinuation of the beta-adrenergic blocking agent B) Prothrombin and activated partial thromboplastin times C) Pulmonary function tests D) Adenosine stress echocardiography E) No further measures

15. A 33-year-old woman undergoes arthroscopy to repair cartilage in the right knee. Nine days later, she consults you because of swelling and tenderness of the right calf. Symptoms have increased during the past two days. She has no personal history of deep vein thrombosis or family history of thromboembolic disease. The patient is afebrile, and vital signs are normal. The right calf is swollen (circumference is 3 cm larger than the left) and tender. Duplex scanning discloses a thrombus involving the calf and superficial femoral vein. Which of the following should you do next?

A) Prescribe rest, leg elevation, and a non-steroidal anti-inflammatory agent and perform serial duplex scans; treat if thrombus progresses.

B) Begin warfarin. C) Begin low-molecular-weight heparin and warfarin. D) Order ventilation–perfusion lung scan. E) No intervention.

16. A 25-year-old man with AML is admitted to the ICU from the bone marrow transplant unit where he had received an allogeneic transplant 13 days ago. His bone marrow is recovering. His leukocyte count had risen from 0.1 to 0.5x109/l. within hours, he become increasingly dyspneic and hypoxic and chest x-ray shows diffuse bilateral infiltrate. He is febrile, tachypneic and tachycardic. BP is 144/94 mmHg and O2 saturation is 88% on 100% oxygen. He is in distress with bilateral scattered crackles. Cardiac examination reveals no gallops or murmurs. His hematocrite is 26%, platelet count is 26x109/l and he had no sputum. Blood cultures are drawn followed by empirical broad spectrum antibiotic therapy. What is the most appropriate next step in this patient’s management?

A) Intravenous corticosteroids and platelet transfusion. B) Continuous positive airway pressure C) Bronchoscopy with bronchoalveolar lavage. D) Noninvasive positive pressure ventilation. E) Intubation.

17. All the following agents are approved as monotherap y in treating neutropenic fever EXCEPT:

A) Imipenem. B) Meropenem. C) Cefepime. D) Gentamicin. E) Ceftazidime.

Page 6: DR. FAHMI KHAN:  · 2017-10-18 · 1. A 71-year-old man undergoes a partial colectomy for colon cancer. He develops temperature of 38.6º C on the third post operative day and rigors

6

18. A 68-year-old man is admitted to the intensive care unit with spontaneous retroperitoneal bleeding and hypotension. He has a medical history of hypertension, diabetes mellitus, and chronic kidney disease stage III. His medications include lisinopril, amlodipine, sitagliptin, and glimepiride. On initial presentation, he is in pain and has a blood pressure of 70/40 mmHg with a heart rate of 132 beats/min. His hemoglobin on admission is 5.3 g/dL and hematocrit is 16.0%. His coagulation studies demonstrate an aPTT of 64 seconds and a PT of 12.1 seconds (INR 1.0). Mixing studies (1:1) are performed. Immediately, the aPTT decreases to 42 seconds. At 1 hour, the aPTT is 56 seconds, and at 2 hours, it is 68 seconds. Thrombin time and reptilase time are normal. Fibrinogen is also normal. What is the most likely cause of the patient's coagulopathy?

A) Acquired factor VIII deficiency B) Acquired factor VIII inhibitor C) Heparin D) Lupus anticoagulant E) Vitamin K deficiency

19. A patient with metabolic acidosis, reduced anion gap, and increased osmolal gap is most likely to have which of the following toxic ingestions?

A) Lithium. B) Methanol. C) Oxycodone. D) Propylene glycol. E) Salicylate.

20. A 37-year-old male respiratory therapist is seen in your office 1 day after developing fever, rigors, generalized muscle aches, and mild respiratory symptoms. He is otherwise in excellent health. The patient mentions that he had not received an influenza vaccination in the year. Physical examination is normal except for a temperature of 39.4 °C (103.0 °F) and coryza. Influenza A has been documented in your community, and you have seen several patients with similar symptoms this week. Which of the following is most appropriate for mana ging this patient?

A) Obtain a chest radiograph and blood cultures B) Obtain a nasopharyngeal culture for influenza and treat only if the result is positive C) Prescribe oseltamivir OR zanamivir. D) Prescribe a fluoroquinolone antibiotic E) Administer an intramuscular or intravenous dose of ceftriaxone and prescribe

azithromycin 21. A 55 year old patient with acute necrotizing pancreatitis has been in the Intensive Care unit with multiple organ failure for approximately 4.5 weeks. Although his clinical situation improved over the last few days, the patient now deteriorates. There is fever up to 104 °F, a rising CRP of 398 and white blood cell count of 27 x 109/L. A contrast enhanced CT demonstrates a large, heterogeneous, walled off, peripancreatic collection with gas inside. What is the preferred treatment strategy at this point?

A) Conservative treatment B) Image-guided percutaneous or endoscopic catheter drainage C) Primary open necrosectomy D) Primary minimally invasive surgical necrosectomy E) Video assisted retroperitoneal debridement

Page 7: DR. FAHMI KHAN:  · 2017-10-18 · 1. A 71-year-old man undergoes a partial colectomy for colon cancer. He develops temperature of 38.6º C on the third post operative day and rigors

7

22. How would you differentiate between hemolytic anemi a and anemia of chronic blood loss?

A) Decrease haptoglobin B) Increased reticulocyte count C) Megaloblastosis D) Polychromasia E) Raised ferritin

23. In gastro-esophageal reflux disease, which of t he following is true?

A) The diagnosis is reliably excluded by a normal endoscopy B) Symptoms correlate well with the severity of the endoscopic findings C) PPI control the symptoms well but are ineffective at healing esophagitis D) After completion of a satisfactory course of PPI symptomatic relapse is uncommon E) Long term proton pump inhibitor treatment reduces the likehood of stricture recurrent

after successful dilatation. 24. A 70-year-old man presents to the emergency department after a 1-min episode of loss of consciousness while eating dinner. Potential cardiac causes of this syncopal episode include all of the following EXCEPT

A) Bradycardia B) Tachycardia C) Aortic stenosis D) Aortic regurgitation E) Ischemia

25. A 65-year-old man presents with 4 months history of swallowing difficulties (worse with liquids than solids). He also complains of nasal regurgitation, coughing and choking episodes during meals and slight dysarthria. He lost 1 stone over the last 8 weeks. Which of the following investigations is the most appropriate fo r this case?

A) Gastroscopy B) Barium swallow C) CXR. D) Tumour markers E) Acetyl choline receptors antibody

26. A 15-year-old old boy presents with a sore throat and macroscopic hematuria. What would light microscopy of a kidney biopsy most likely show?

A) Crescentic glomerulonephritis B) Collapsed glomeruli C) Mesangial hypercellularity D) Segmental sclerosis E) Normal tissue

27. In diabetic neuropathy you can find each of the following EXCEPT?

A) Bradycardia B) Urine retention C) Impotence D) Diarrhea at night E) Foot ulcer

Page 8: DR. FAHMI KHAN:  · 2017-10-18 · 1. A 71-year-old man undergoes a partial colectomy for colon cancer. He develops temperature of 38.6º C on the third post operative day and rigors

8

28. A patient undergoing assessment for potential cardiac transplantation has a right heart catheterization performed for the evaluation of pulmonary hypertension. The hemodynamic review of his right-sided pressures reveals a pulmonary vascular resistance (PVR) of 4 Wood units (320dynes-sec/cm5). What should be the next step in this patient's eval uation?

A) Proceed with heart transplantation B) Evaluate for combined heart and lung transplantation C) Discontinue heart transplant evaluation as the patient is no longer a candidate D) Vasodilator challenge with nitric oxide E) Add bisoprolol

29. A 42-year-old woman presents with acute onset of headache and neck pain vertigo, nausea and vomiting. Neurological examination reveals left nystagmus, left Horner syndrome and absent left gag reflex as well as left appendicular ataxia and anesthesia to pin prick in the left face and right arm and leg. Which of the following is the most likely diagnosis ?

A) MCA B) PCA C) ACA D) dissection of aorta E) Vertebral artery dissection

30. A 38-year-old woman is evaluated for new onset fatigue, Joint pain and jaundice. She has a history of hypothyroidism and her only medications are levothyroxine and MV. She has never used illicit drugs and does not drink alcohol. Her mother has SLE. O/E the patient is afebrile, BP 130/75, the pulse rate 80 and the RR 14. BMI 25. The sclera is icterus. The rest of the examination is normal. LABS: CBC normal, AST 890, ALT 75, ALP 120, t. bilirubin 6 mg/dl, direct 3.6, ANA 1:80, Anti SMA titer 1/640, AMA, negative, viral serologic test are negative Which of the following is the most likely diagnosis ?

A) Primary sclerosing cholangitis B) Acute cholecystitis C) Drug induced liver injury D) Primary biliary cirrhosis E) Autoimmune hepatitis

31. In a patient with vasculitis, the finding of serum antineutrophil cytoplasmic autoantibodies (ANCAs) that react by immunofluorescence staining in a perinuclear pattern (P-ANCAs) is most suggestive of which one of the following diseases:

A) Classic polyarteritis nodosa B) Wegener's granulomatosis C) Churg-Strauss syndrome D) Microscopic polyangiitis E) Giant cell arteritis

32. Which SINGLE clinical feature has the most specificity in differentiating Pseudomonas aeruginosa sepsis from other causes of severe sepsis in a hospitalized patient?

A) Ecthyma gangrenosum B) Hospitalization for severe burn C) Profound bandemia D) Recent antibiotic exposure E) Recent mechanical ventilation for >14 days

Page 9: DR. FAHMI KHAN:  · 2017-10-18 · 1. A 71-year-old man undergoes a partial colectomy for colon cancer. He develops temperature of 38.6º C on the third post operative day and rigors

9

33. Which of the following statement is true of Mycobac terium tuberculosis? A) non-sputum producing patients are non-infectious B) a positive tuberculin test indicates active disease C) lymph node positive disease requires longer treatment than pulmonary disease D) in pregnant women treatment should not be given until after delivery E) pyrazinamide has high activity against active extracellular organisms

34. You are managing a patient who is hypotensive, refractory to intravenous fluids. You decide to use a pure alpha-adrenergic agent, which of the following is the appropriate one :

A) Dopamine. B) Dobutamine. C) Amrinone D) Isoproterenol E) Phenylephrine.

35. Which of the following infections is least likely t o cause myocarditis? A) Coxsackie virus. B) Diphtheria. C) Chagas Disease. D) Syphillis. E) Toxoplasmosis.

36. A 36-year-old white female presents to the emergency department with palpitations. Her pulse rate is 180/min. An EKG reveals a regular tachycardia with a narrow complex QRS and no apparent P waves. The patient fails to respond to carotid massage or to two doses of intravenous adenosine, 6 mg and 12 mg. The most appropriate next step would be to administer intravenous:

A) amiodarone B) digoxin C) flecainide D) propafenone E) verapamil

37. A 58-year-old white male comes to your office for follow-up after a recent bout of acute bronchitis. He reports having a productive cough for several months. He gets breathless with exertion and notes that every time he gets a cold it "goes into my chest and lingers for months." He has been smoking for 30 years. A physical examination is negative except for scattered rhonchi. A chest radiograph done 4 months ago at an urgent care visit was negative except for hyperinflation and flattened diaphragms. Which one of the following would be best for making the diagnosis?

A) A chest radiograph B) CT of the chest C) Peak flow measurement D) Spirometry E) A BNP level

38. Which biochemical test is an early predictor of bil iary etiology of acute pancreatitis? A) Serum elastase B) Lipase C) Bilirubin D) G-Glutamyltransferase E) None of the above

Page 10: DR. FAHMI KHAN:  · 2017-10-18 · 1. A 71-year-old man undergoes a partial colectomy for colon cancer. He develops temperature of 38.6º C on the third post operative day and rigors

10

39. A 30-year-old renal transplant recipient presented with non-Hodgkin's lymphoma. Which virus is most likely to be of etiological significa nce?

A) Adenovirus B) Cytomegalovirus C) Epstein Barr virus D) Herpes simplex type 1 E) Varicella-zoster

40. Which one of the following is true regarding respir atory syncytial virus infection? A) It only affects children B) Corticosteroids should be a routine part of treatment C) The diagnosis is usually based on positive serology D) It is rarely associated with bacterial co-infection E) It is a single-stranded enveloped DNA Paramyxovirus.

41. Which one of the following is most likely to be see n with diastolic dysfunction? A) A dilated left ventricle B) A preserved ejection fraction C) Aortic insufficiency D) Pericardial effusion E) Myocarditis

42. A 30-year-old male presents to your office because he thinks he may be suffering from alcohol withdrawal. He was dependent on alcohol for at least 10 years and has completed treatment programs twice. He had been abstinent for over a year until he began drinking heavily after his wife filed for divorce 2 weeks ago. A friend found him in a bar last night and has kept him from consuming alcohol for the past 12 hours. The patient is now nauseated, miserable, restless, shaky, and sweating, and says he can feel his heart pounding. He has not had any seizures or episodes of delirium tremens. His temperature is 37.5°C (99.6°F), pulse rate 100/min, and blood pressure 150/92 mm Hg. His palms are moist and he has a mild tremor on arm extension. He is oriented but cannot perform serial additions. A CBC, basic metabolic panel, and urine drug screen are normal. You decide that outpatient treatment would be appropriate. Which one of the following alcohol withdrawal manag ement options is supported by the best evidence?

A) Thiamine and magnesium B) Carbamazepine C) Phenytoin D) Chlordiazepoxide E) Clonidine

43. A 58-year-old man presents with fatigue. His physical examination is normal except for the presence of splenomegaly. CBC discloses hematocrit, 29%; platelet count, 90,000/L; WBC, 2700/L; and essentially normal red cell morphology (differential 12% monocytes, 12% granulocytes, and 76% lymphocytes). A bone marrow aspirate and biopsy were performed. The aspirate was dry and the biopsy is pending. Based on the available information, the most likely diagnosis in this case is:

A) Chronic lymphocytic leukemia B) Hairy cell leukemia C) Chronic myeloid leukemia D) Myelofibrosis E) Multiple myeloma

Page 11: DR. FAHMI KHAN:  · 2017-10-18 · 1. A 71-year-old man undergoes a partial colectomy for colon cancer. He develops temperature of 38.6º C on the third post operative day and rigors

11

44. A 41-year-old African man has a history of multiple episodes of sudden onset of severe abdominal pain and back pain lasting for hours. Each time this happens, his peripheral blood smear demonstrates numerous sickled erythrocytes. A haemoglobin electrophoresis shows 94% Hgb S, 5% Hgb F, and 1% Hgb A2. He now has increasing pain in his right groin radiating to the anterior aspect of the thigh and to the knee. His temperature was 38°C and examination of his hip revealed pain on internal rotation. A radiograph reveals irregular bony destruction of the femoral head. The most likely organism to be responsible for thes e findings is?

A) Candida albicans B) Clostridium perfringens C) Group B streptococcus D) Salmonella species E) Yersinia pestis

45. A 35 year old man referred for evaluation of long standing hypertension. Examination shows grade 3/6 systolic ejection murmur. Electrocardiography shows left ventricular hypertrophy with strain. ECHO suggests bicuspid aortic valve (BAV) without significant stenosis or regurgitation. What is the next most appropriated step?

A) Refer the patient for magnetic resonance imaging of thoracic aorta B) Treatment with B-adrenergic blocker C) Reassurance and follow up in 5 years D) Genetic counseling E) Native BAV replacement by a prosthesis

46. In a patient who had kidney transplantation, trimethoprim sulfmethaxazole is protective of all of the following microorganisms EXCEPT:

A) Pneumpcystis carinii B) Bacterial urinary tract infection C) Toxoplasma gondii D) Rickettesia rickettsi E) Nocardia brasiliensis

47. A 15 years old male presented with 2 days history of fever and right knee swelling following upper respiratory tract infection. O/E showed right knee to be warm with moderate effusion, tenderness and surrounding redness, 2 similar attacks have been noted in the last 2 years. The best approach in the above patient is:

A) Start penicillin, ASA, re evaluate in 2 day after routine blood test are ready. B) Blood, throat culture, start NSAIDS, and re evaluate 2 days after routine blood test are

ready C) Obtain synovial fluid for analysis and start wide spectrum antibodies. D) Obtain synovial fluid for analysis and start NSAIDS. E) Admit to hospital; obtain routine test, blood and urine culture and synovial fluid analysis.

48. Choose the correct statement about treating an emer gency department patient who presents with diabetic ketoacidosis (DKA):

A) The average fluid deficit in DKA is 4–6 L. B) Initial IV fluid replacement is best done with hypotonic fluids. C) A negative test for serum ketones in the ED does not exclude the diagnosis of DKA. D) Because of severe hemoconcentration, most patients with DKA have a dangerously high

serum potassium level. E) Intravenous bicarbonate will rapidly and safely correct the severe metabolic acidosis.

Page 12: DR. FAHMI KHAN:  · 2017-10-18 · 1. A 71-year-old man undergoes a partial colectomy for colon cancer. He develops temperature of 38.6º C on the third post operative day and rigors

12

49. A 46-year-old woman with a history of Grave’s disease is brought to the ED several hours after outpatient plastic facial surgery. She is delirious and agitated. Vital signs: temperature, 39.2◦C; BP, 140/60 mm Hg, respiratory rate, 35/min; physical examination is otherwise unremarkable. Medications include propylthiouracil and a diuretic. A WBC is 10x109/L with no left shift. Findings from other examinations, including CXR and urinalysis, are negative. Treatment at this time should include:

A) Aspirin, rather than acetaminophen, to treat hyperpyrexia. B) Calcium gluconate infusion to prevent rebound hypocalcemia. C) Propranolol 1–2 mg slow IV push. D) Ice baths. E) Gentamicin 100 mg and ampicillin 1 g IV.

50. A 50-year-old woman with a history of hypertension complains of 2 weeks of numbness and tingling in her extremities associated with progressive weakness. She takes amlodipine for blood pressure control. Laboratory results: sodium, 144 mEq/L; potassium, 1.9 mEq/L; bicarbonate, 15 mEq/L; chloride, 107 mEq/L; BUN, 17 mEq/L; creatinine, 0.6 mg/dL. Urinalysis shows: pH, 6.7, with no blood or ketones. The most likely associated symptom in this setting is:

A) Hypoaldosteronism. B) Fanconi syndrome. C) Nephrocalcinosis. D) Laxative abuse. E) Furosemide abuse.

51. A 25-year-old athletic with no family history of ischemia heart disease presents with a 3-months history of exertional chest pain, the pain is retrosternal and described as a dull ache, it tends to persist for up to half an hour after cessation of exercise. The following are true EXCEPT:

A) Cardiac disease should be excluded by a combination of risk assessment and exercise ECG

B) An ECHO should be performed C) Prolonged ambulatory esophageal PH measurement is an appropriate investigation D) An acid perfusion test should be performed E) The patient should be advised to cease athletic sports

52. A 49-years-old woman experiences fatigue and weakness. Her history reveals only severe headache for past 20 years. Physical examination reveals a shallow, chronically ill appearing women, the remainder of physical examination is negative. lab test reveal Hb9gm the urinalysis reveal a trace of proteins 10 to 12 white cells and a rare white cast , urine culture is sterile , BUN and creatinine are 40 mg /dl (14.3mmol/l) and 2.4mg/dl ( 212umol/l) respectively. What is the most likely diagnosis of this patient?

A) Chronic glomerulonephritis B) Inactive pyelonephritis C) Interstitial nephritis D) Multiple myeloma E) SLE

Page 13: DR. FAHMI KHAN:  · 2017-10-18 · 1. A 71-year-old man undergoes a partial colectomy for colon cancer. He develops temperature of 38.6º C on the third post operative day and rigors

13

53. A 36-year-old previously healthy woman, hospitalized because of the recent onset of progressive dyspnea, and lung infiltrates, she now has diffuse myalgia and skin rash over the back and face. The ESR is 86 mm/h. Lung biopsy showed bronchiolitis obliterans with organizing pneumonia (BOOP). The following disorders are likely to be associated with the above findings EXCEPT:

A) Rheumatoid Arthritis B) Polymyositis-dermatomyositis C) Alpha1-antitrypsin deficiency D) Amiodarone lung toxicity E) Esinophilic pneumonia

54. Regarding hospital- acquired infections, all of the following are correct EXCEPT:

A) Current preventive measures cannot prevent many nosocomial infections B) Staph aureus transmitted from hospital workers are more-antibiotic resistant C) Urinary tract infection is the most common hospital acquired infection D) The most serious infection is pneumonia E) In surgical wound infections staph aureus and Staph epidermidis are the most commonly

isolated pathogens 55. All of the following are true about heat stroke EXCEPT:

A) Core temperature above 41.5o C is expected B) Acute renal failure is a recognized feature C) Hypokalemia occurs in this condition D) Rhabdomyolysis is a feature E) DIC can occur in this condition

56. A 50 year old man, a 100 pack smoker, develops progressive weakness. Chest radiography shows mediastinal widening since an examination 6 months earlier. The serum K is 2.3 mEg/L. Which one of the following is FALSE about this pati ent?

A) Chest CT is likely to show a medial/hilar mass B) EMG will show decreasing muscle strength on repetitive stimuli C) Serum ACTH level may be increased D) Hyponatremia is a potential complication E) Surgery is not a therapeutic option

57. A 65-year-old male is admitted to hospital with 2 hours retrosternal pain finally relieved with morphine. He has history of similar pain chest pain 6months ago and at that time myocardial infarction was ruled out and cardiac catheterization was normal. He has occasional heartburn without regurgitation or dyspepsia. His brother died of myocardial infarction. He has had neither hypertension nor diabetes. On examination, obese and resting comfortably. Cholesterol 240 mg/dl (6.2 mmol/L). Other lab studies are normal. Chest x-ray, ECHO, abdominal ultrasound are normal. Highest yield procedure to determine cause of chest pain is:

A) Upper endoscopy B) Bernstein test C) 24hr esophageal pH monitoring D) Esophageal motility E) Edrophonium provocation test

Page 14: DR. FAHMI KHAN:  · 2017-10-18 · 1. A 71-year-old man undergoes a partial colectomy for colon cancer. He develops temperature of 38.6º C on the third post operative day and rigors

14

58. A 78-year-old woman is admitted to the hospital because of a fever, productive cough, and a chest x-ray demonstrating right lower lobe consolidation. Her past medical history is significant for seasonal allergies. She has been taking estrogen/progesterone replacement since menopause 19 years ago and occasional acetaminophen for headaches. The patient lives alone at her home and she does not drink alcohol or smoke. Review of systems is significant for weakness attributed to “old age". On the day prior to discharge, a repeat chest x-ray shows the pneumonia to be resolving. An incidental note is made of severe osteoporosis involving all of the bones visualized on the film. Vital signs are temperature 38.8 C (101.8 F), blood pressure 100/50 mm Hg, pulse 90/min, and respirations 10/min. Physical examination is significant only for decreased breath sounds at the right lung base. The patient is neurologically intact and wants to return home. Laboratory studies show a leukocyte count 15x109/L, hematocrit 28%, and platelets 150x109/L. The next step in the management of this patient is to:

A) Discharge her and do a bone marrow biopsy as an outpatient B) Discharge her and send her for a bone scan as an outpatient C) Discharge her and order serum protein electrophoresis as an outpatient D) Do a bone marrow biopsy before discharge E) Order a bone scan and serum protein electrophoresis before discharge

59. A patient who was diagnosed with rheumatoid arthritis 2 years ago, now on prednisolone 5mg/day , methotrexate 15mg/wk, and naproxen 500mg twice daily, comes to your office complaining of severe dryness in the mouth , he has checked his fasting blood sugar and found it to be normal , which statement is correct in regards to this pt co nditions ?

A) This is relatively common complication of rheumatoid arthritis B) Stopping naproxen will improve the patient symptoms C) Adding Hydroxychloroquine usually results in prompt relive of the patient complaints D) Increasing the dose of steroid usually results in improvement of this condition E) Decreasing the dose of Methotrexate or changing to another disease modifying agent

usually causes quick resolution of this problem

60. Coumadin-induced skin necrosis is occasionally asso ciated with the institution of oral anticoagulants in patients with:

A) Antithrombin III deficiency B) Protein C deficiency C) Factor VIII deficiency D) Plasminogen deficiency E) Dysfibrinogenemias

61. A 55-year-old man with a history of alcoholism complains of more than a month of malaise, low-grade fever, and a productive cough with greenish sputum tinged with blood. Examination shows periodontal disease with bad breath and clubbing of fingers. On chest x-ray, there is a 2 cm cavity with an air–fluid level in the posterior segment of the right upper lobe. Sputum smear shows many neutrophils and a variety of bacteria. Appropriate treatment includes:

A) Isolate the patient and initiate a four-drug antituberculosis treatment. B) Start intravenous administration of clindamycin. C) Refer the patient to a dentist for periodontal care. D) Schedule a bronchoscopy for the next day. E) Start administration of methicillin and tobramycin.

Page 15: DR. FAHMI KHAN:  · 2017-10-18 · 1. A 71-year-old man undergoes a partial colectomy for colon cancer. He develops temperature of 38.6º C on the third post operative day and rigors

15

62. A 19-year-old woman is brought to the emergency department by her roommate, who reports the patient is “not acting right.” She has a history of depression for which she is prescribed fluoxetine. Her friend reports that she was well and without complaints until a few hours prior to arrival when she took some ecstasy. Vital signs: temperature 105°F, heart rate 125/min, respiratory rate 20/min, blood pressure 140/80 mm Hg, and pulse oximetry of 99% on room air. Her physical examination is notable for diaphoresis, confusion, and disorientation. She also has lower extremity hypertonicity and clonus. Appropriate treatment includes administration of:

A) Charcoal. B) Acetaminophen. C) Glucose. D) Dantrolene and benzodiazepines. E) Cyproheptadine and benzodiazepines.

63. You choose to use intravenous magnesium sulfate to treat a patient with status asthmaticus, the most important toxicity sign you have to watch for is:

A) Respiratory depression. B) Diarrhea. C) Pruritus. D) Hyperreflexia. E) Dry mouth.

64. A 62-year-old woman presents with an altered mental status. She is responsive to painful stimuli and has a Glasgow Coma Scale score of 11. Blood pressure is 100/60 mm Hg and heart rate is 100/min. Physical examination reveals poor skin turgor and dry mucus membranes. ECG shows QT interval shortening. Laboratory findings are serum calcium 14.2 mg/dL (3.55 mmol/L), serum phosphorus 2.9 mg/dL (0.93 mmol/L), serum potassium 3.9 mEq/L, and creatinine 1.9 mg/dL (167.9 µmol/L). Your initial treatment should include:

A) Magnesium sulfate 2 gm slow IV push. B) Pamidronate 60 mg IV. C) Calcitonin 4 IU/kg. D) Furosemide 60 mg IV after isotonic saline resuscitation. E) Immediate hemodialysis.

65. A 54-year-old man with past medical history of HIV, hypertension, and tuberculosis is brought to the emergency department by a family friend after he is found seizing on the floor of his living room. The seizures persist despite the administration of appropriate doses of benzodiazepines. Serum glucose is normal. The next agent you should give is:

A) Phenytoin. B) Barbiturates. C) Pyridoxine. D) Benzodiazepines. E) Propofol.

Page 16: DR. FAHMI KHAN:  · 2017-10-18 · 1. A 71-year-old man undergoes a partial colectomy for colon cancer. He develops temperature of 38.6º C on the third post operative day and rigors

16

66. A 52-year-old man complains of severe right knee pain. He recalls no trauma, but attended a wine tasting party last weekend. He denies past medical history and is afebrile with normal vital signs. Physical examination reveals a swollen, red, painful right knee. Radiograph shows an effusion, but no bony erosions. You perform arthrocentesis and obtain 27 mL of cloudy straw-colored fluid, which you send to the laboratory for studies. The cell count is 50,000 WBCs/mm3 with 85% PMNs, glucose 120 mg/dL (serum 130), and protein 3.5 g/dL. The Gram stain shows numerous WBCs, but is negative for organisms. Evaluation for crystals shows numerous needle-shaped crystals with negative birefringence. A good treatment plan would be:

A) Oral colchicine 0.5 mg every hour until symptoms abate, GI toxicity develops or the maximum dose of 6 mg in 24 hours is reached.

B) Subcutaneous colchicine 1 mg plus oral probenecid 250 mg bid for 1 week. C) Sublingual colchicine 2 mg followed by oral steroids tapered over 5 days. D) Intra-articular triamcinolone injection. E) Oral allopurinol 500 mg tid for 1 week.

67. A 35-year-old woman presents to her gynecologist with complaints of burning on urination for the past 2 days. Dipstick test of her urine demonstrates marked positivity for leukocyte esterase, but no reactivity for nitrite. Urine culture later grows out large numbers of organisms. Which of the following bacteria is most likely caus ative agent for this infection?

A) Enterobacter sp. B) Enterococcus faecalis. C) Escherichia coli. D) Klebsiella pneumoniae. E) Pseudomonas aeruginosa.

68. An 18 year old woman is admitted to the hospital two days after becoming confused, disoriented, and unsteady in gait. During the past three months she has been depressed and has declined food, except for some cake, cookies and liquids. She has lost weight. She appears thin and undernourished and is disoriented to time and place. She reports having double vision, neither eye abducts normally. Gait is unsteady though the limbs are strong. The liver and spleen are not enlarged. Which of the following would be most appropriate in itially?

A) NG and high calorie diet B) Intravenous 50% glucose. C) Intravenous thiamine. D) Intravenous methylprednisolone. E) Intravenous Vitamin k.

69. A 60-year-old man with a past history of controlled hypertension presents with acute onset weakness of his left arm that resolved over 12 hours. He had suffered two similar episodes over the last three months. Examination reveals a blood pressure of 132/82 mmHg and he is in atrial fibrillation with a ventricular rate of 85 per minute. CT brain scan is normal. What is the most appropriate management?

A) Amiodarone. B) Digoxin. C) Aspirin. D) Dypyridamole. E) Lifelong anticoagulation.

Page 17: DR. FAHMI KHAN:  · 2017-10-18 · 1. A 71-year-old man undergoes a partial colectomy for colon cancer. He develops temperature of 38.6º C on the third post operative day and rigors

17

70. A 35-year-old healthy woman has a faint systolic murmur on physical examination. An echocardiogram is performed, and she is found to have a bicuspid aortic valve. In explaining the meaning of this finding to her, the most approp riate statement is that?

A) An aortic valve replacement is eventually likely to be required. B) Other family members are likely to have the same condition. C) She should be treated with a cholesterol lowering agent. D) The problem resulted from past injection drug usage. E) This is one manifestation of an underlying autoimmune condition.

71. In a survey of 100 patients, 30 had ascites of which 25 had alcoholic cirrhosis. 10 other patients who had no ascites did have alcoholic cirrhosis. Which of the following statement is correct?

A) Positive predictive value is 25/(25+5). B) Positive predictive value is 25/(25+10). C) Sensitivity is 25/(25+5). D) Specificity is 25/(25+5). E) Specificity is 5/(25+5).

72. All of the following are indications to treat autoimmune hepatitis EXCEPT A) AST more than 10 folds normal. B) AST more than 5 folds with gamma globulins more than 2 fold normal. C) Bridging necrosis on histology. D) Minimally active cirrhosis. E) Severe symptoms.

73. A 26-year-old woman presented in acute shock at 35 weeks of pregnancy with profuse vaginal bleeding. She had suffered two previous miscarriages. She had a pulse of 110 beats per minute, blood pressure of 110/84 mmHg and no fetal heart sounds were audible. Investigations: Haemoglobin concentration 9.5g/dL (11.5 – 16.5), platelet count 66 X 109/L (150 – 400), prothrombin time 21 s (11.5 – 15.5), activated partial thromboplastin time (APTT) 52 s (30 – 40), fibrinogen concentration 0.5 g/L (2 – 4). What is the most appropriate next step in her manag ement?

A) Antithrombin III infusion. B) Fibrinogen replacement infusion (cryoprecipitate). C) Intravenous heparin. D) Platelet transfusion. E) Transfusion of two units group O Rhesus D negative blood.

74. An 81 -year-old man admitted with a stroke becomes increasingly drowsy after receiving nasogastric feeding for five days. Which biochemical abnormality is the most likely ca use of his drowsiness?

A) Hyperglycemia. B) Hypermagnesemia. C) Hypernatremia. D) Hypocalcemia. E) Hypophosphatemia.

Page 18: DR. FAHMI KHAN:  · 2017-10-18 · 1. A 71-year-old man undergoes a partial colectomy for colon cancer. He develops temperature of 38.6º C on the third post operative day and rigors

18

75. A 40-year old lady presents to clinic complaining of an 18 month history of dorsoradial wrist pain. She is a keen tennis player. On examination she has tenderness localized to the dorsoradial aspect of the wrist and passive motion of the thumb causes crepitus in the same region. Finkelstein’s test is positive. What is the likely diagnosis?

A) Carpal tunnel syndrome. B) De Quervain’s tenosynovitis. C) Golfer's elbow. D) Tennis elbow. E) Ulnar tunnel syndrome.

76. A 19-year-old woman presents to the clinic having had 5 blackouts over the last year, all while she is standing up. She gets warnings of blurred vision, nausea, feeling hot. She had been witnessed twice to have jerking of all limbs while she is unconscious. The attacks last 30-60 seconds. She recovers quickly after the attacks. She has never bitten her tongue or sustained any injuries. Physical examination and an ECG are normal. Her grandmother and sister suffer from epilepsy. Which of the following investigations is the most a ppropriate?

A) EEG. B) 24 hour ECG recording. C) CT brain. D) ECHO. E) Tilt table test.

77. The medication of choice to treat a patient in tors ades de pointes is :

A) Epinephrine. B) Flecainide. C) Calcium gluconate D) Magnesium sulfate. E) Procainamide.

78. You are caring for a patient with severe COPD who will require intubation and mechanical ventilation. To limit or prevent the development of dynamic hype rinflation, your next step should be:

A) Increase the minute ventilation. B) Increase the expiratory flow rates of the ventilator. C) Increase the I–E ratio of the ventilator. D) Reduce the FIO2. E) Intubate with a small diameter endotracheal tube.

79. The advantages of endoscopy over barium radiography in the evaluation of dysphagia include all of the following EXCEPT :

A) Ability to intervene as well as diagnose. B) Ability to obtain biopsy specimens. C) Increased sensitivity for the detection of abnormalities identified by color, e.g., Barrett's

metaplasia. D) Increased sensitivity for the detection of mucosal lesions. E) No meaningful risk to procedure.

Page 19: DR. FAHMI KHAN:  · 2017-10-18 · 1. A 71-year-old man undergoes a partial colectomy for colon cancer. He develops temperature of 38.6º C on the third post operative day and rigors

19

80. A 23 year old single woman referred with 3-month history of weight loss and heat intolerance. On examination pulse is 120 beat/min regular, blood pressure 120/72 mmHg, weight 58 kg, height 165 cm. she had diffuse goiter with bilateral exophthalmos. Investigations: serum free T4 3.9 ng/dl (N: 0.9-2.0), serum TSH 0.001 mu/l (N: 0.4-4.0), TSH receptor antibody 8 U/L(N:<2). She was started on methimazole 30 mg/day. Six weeks later she developed severe migratory arthralgia associated with the presence of palpable purpura over both legs and dermal infarcts in the finger tips. Laboratory studies shows ESR 54 mm/h, urine showed microscopic hematuria, ANCA titer was 1:640, antimyeloperoxidase antibodies 112U/l (N:0-9), antiproteinase 3 antibodies 6.4 U/l (N:0-3.5). The best action is:

A) Continue methimazole and add NSAID. B) Continue methimazole and add prednisolone 40 mg per day. C) Substitute propylthiouracil for methimazole. D) Refer the patient for definitive therapy with radioactive iodine. E) No intervention.

81. A 47-year-old man is evaluated for 1 year of recurrent episodes of bilateral ear swelling. The ear is painful during these events, and the right ear has become floppy. He is otherwise healthy and reports no illicit habits. He works in an office and his only sport is tennis. On examination, the left ear has a beefy red color, and the pinna is tender and swollen; the earlobe appears minimally swollen but is neither red nor tender. Which of the following is the most likely explanati on for this finding?

A) Behçet's syndrome. B) Cogan's syndrome. C) Hemoglobinopathy. D) Recurrent trauma. E) Relapsing polychondritis.

82. A 45-year-old man comes to his doctor's office with lower back pain. The pain started suddenly after he was lifting a couch as he and his wife were redecorating their living room. The pain radiates down the posterior right thigh to his right ankle. It is worse when he moves. On examination, he has difficulty performing strength testing because he is in pain (it hurts when he moves). The pain is brought out with right straight leg raises. He has a diminished right ankle jerk reflex. What is the most likely diagnosis?

A) Right L3-4 radiculopathy. B) Left L4-5 radiculopathy. C) Right L4-5 radiculopathy. D) Right L5-S1 radiculopathy. E) Left L3-4 radiculopathy.

83. A 46-year-old woman with a 10-year history of primary progressive multiple sclerosis is admitted to the hospital for surgical debridement and wound management of a sacral pressure ulcer. The patient is bedbound and is cared for at home by her husband. Her medications include sertraline, baclofen, and oxybutynin. The remainder of the medical history is noncontributory. On physical examination, she is thin, in no distress, and has normal vital signs. There are contraction deformities of the lower extremities, and a 6- × 8-cm sacral ulcer that extends to the fascia, with minimal purulent exudates and no evidence of cellulitis. Laboratory studies include a hematocrit of 34%, leukocyte count of 15,000/µL (15 × 109/L) with 80% neutrophils, and a platelet count of 425,000/µL (425 × 109/L). Subcutaneous prophylactic

Page 20: DR. FAHMI KHAN:  · 2017-10-18 · 1. A 71-year-old man undergoes a partial colectomy for colon cancer. He develops temperature of 38.6º C on the third post operative day and rigors

20

unfractionated heparin is administered. Six days after initiation of heparin, her platelet count decreases to 210,000/µL (210 × 109/L), and on the ninth day after therapy, the platelet count has decreased to 95,000/µL (95 × 109/L). Which is the most appropriate next step in the mana gement of this patient?

A) Discontinue unfractionated heparin. B) Discontinue sertraline, baclofen, and oxybutynin. C) Discontinue unfractionated heparin and begin therapy with a direct thrombin inhibitor. D) Discontinue unfractionated heparin and begin low-molecular-weight heparin. E) Discontinue unfractionated heparin and begin novel oral anticoagulants.

84. A 52 year old female presents with tiredness. There are no specific abnormalities noted on examination, but investigations reveal a T4 of 21.1 (NR 9.8 - 23), a T3 of 5.2 pmol/l (NR 3.3 - 5.5) and a TSH of 0.05 mU/l (NR 0.1 - 5 mU/l). Thyroid autoantibody titers are all undetectable. These results suggest a diagnosis of:

A) De Quervain's thyroiditis. B) Sick euthyroid syndrome. C) Solitary toxic nodule. D) Grave's disease. E) Hashimoto's thyroiditis.

85. A 28-year-old man who is known to have Hypertrophic Cardiomyopathy has an out of hospital cardiac arrest and is successfully resuscitated. What is the most appropriate mode of treatment?

A) Alcohol septal ablation. B) Amiodarone. C) Beta blocker. D) Implantable defibrillator. E) Myomectomy.

86. A 69 year old man is treated for chest infection. He has been on a stable dose of warfarin for the last six months as a treatment for atrial fibrillation, with INR recordings between 2-2.5. However, his most recent INR was 5. Which one of the following drugs that has recently been started is likely to be responsible for the increased INR?

A) Clarithromycin. B) Co-dydramol. C) Digoxin. D) Rifampicin. E) Temazepam.

87. A 75 year old man has a history of Chronic Lymphocytic Leukemia. He has had treatment with several courses of chemotherapy and has now been admitted to hospital with pneumonia. His past medical history revealed that he had suffered several previous upper respiratory tract infections over the previous six months. Which of the following components of his immune sys tem is likely to be deficient?

A) Complement. B) Immunoglobulin G. C) Macrophages. D) Mast cells. E) T lymphocytes.

Page 21: DR. FAHMI KHAN:  · 2017-10-18 · 1. A 71-year-old man undergoes a partial colectomy for colon cancer. He develops temperature of 38.6º C on the third post operative day and rigors

21

88. A teenage girl presents with Guillain-Barré syndrome. Her weakness continues to worsen after admission to hospital. Which of the following should be used to monitor he r?

A) Arterial blood gases B) Chest expansion size C) FEV1/FVC ratio D) PEFR E) Vital capacity

89. A 68-year-old woman with atrial fibrillation is admitted for DC cardioversion. The procedure resulted in successful restoration of sinus rhythm. Which one of the following drugs would be most like ly to maintain sinus rhythm following this procedure?

A) Amiodarone. B) Digoxin. C) Diltiazem. D) Sotalol. E) Verapamil.

90. 58-year-old man has an eight months history of watery diarrhea (6-10 motions per day) with urgency but no significant abdominal pain, weight loss or GIT bleeding. Loperamide up to 10 tablets fails to control the diarrhea. Colonoscopy is normal but shows collagenous colitis. Which of the following is the most appropriate ther apy?

A) Increase the dose of leporamide. B) Colloid bismuth salicylate. C) Prednisolone. D) Mesalazine. E) Budesonide.

91. Which of the following methods is best for detectin g early, subclinical interstitial lung disease?

A) Spirometry. B) Arterial blood gas analysis. C) Body plethysmography. D) Diffusing capacity measurement. E) Measurement of blood oxygen-carrying capacity.

92. Which of the following is the most common findi ng in aphasic patients?

A) Alexia. B) Anomia. C) Comprehension. D) Fluency. E) Repetition.

Page 22: DR. FAHMI KHAN:  · 2017-10-18 · 1. A 71-year-old man undergoes a partial colectomy for colon cancer. He develops temperature of 38.6º C on the third post operative day and rigors

22

93. A 37-year-old woman is brought to the ICU after her elective laparoscopic cholecystectomy is complicated by a temperature of 105°F (40.5º C), tachycardia, and systemic hypotension. Examination is notable for diffuse muscular rigidity. Which of the following drugs should be administered immediately?

A) Acetaminophen. B) Dantrolene. C) Haloperidol. D) Hydrocortisone. E) Ibuprofen.

94. A 26 year old patient presents to the clinic for advice. His brother has recently been diagnosed with Hemochromatosis. Which one of the following is the most useful scree ning test?

A) Ferritin. B) HFE gene analysis. C) Ultrasound of the liver. D) Liver iron levels. E) Transferrin saturation.

95. A 65 year old lady has epigastric pain for several months and is referred for endoscopy. Biopsy confirms MALT lymphoma. What is the treatment of choice?

A) Chemotherapy. B) Radiotherapy. C) H. Pylori eradication. D) Interferon. E) Surgery.

96. All the following usually cause apical fibrosis EXCEPT:

A) Ankylosing spondylitis. B) Langerhans granuloma. C) Asbestosis. D) Silicosis. E) Tuberculosis.

97. Paraneoplastic syndromes in small cell lung cancer are associated with all of the following EXCEPT:

A) Hypercalcemia. B) SIADH. C) Cerebellar degeneration. D) Ectopic Cushing syndrome. E) Eaton Lambert syndrome.

98. Which one of the following leukemia’s charactestica lly presents with DIC?

A) Acute monobasic leukemia. B) Acute promyelocytic leukemia. C) Acute lymphoblastic leukemia. D) Chronic myeloid leukemia. E) Chronic lymphocytic leukemia.

Page 23: DR. FAHMI KHAN:  · 2017-10-18 · 1. A 71-year-old man undergoes a partial colectomy for colon cancer. He develops temperature of 38.6º C on the third post operative day and rigors

23

99. A 60-year-old farmer presented with non specific symptoms of chronic cough and progressive dyspnea for the last six years. The patient had interstitial pneumonitis. Chest x-ray revealed reticulonodular infiltrate associated with honey-combing of the lung. Pulmonary function studies showed a restrictive pattern with loss of lung volumes, impaired diffusion capacity, decreased compliance and exercise induced hypoxia. This farmer’s symptoms are most likely secondary to the following environmental hazard:

A) The farmers cheese processing plant. B) The presence of spores of bacillus anthracis in the farmer’s farm. C) The harvesting of crops in rainy weather. D) The presence of asbestos in the water tanks. E) The presence of antigenic detergents.

100. A 54 year old man presents with central crushing chest pain. Examination is normal. 12-lead ECG shows ST segment elevation in leads II, III, aVF and ST depression in V1, V2 and V3. Which coronary artery is occluded?

A) Circumflex. B) Left anterior descending. C) Left main stem. D) Obtuse marginal. E) Right coronary artery.

101. A 54-year-old man is admitted to the intensive care unit with severe pancreatitis. His BMI is 30 or above and he has a prior history of diabetes mellitus. A CT of the abdomen is obtained and shows severe necrotizing pancreatitis. He is presently afebrile. Which of the following medications has been shown t o be effective in the treatment of acute necrotizing pancreatitis?

A) Calcitonin. B) Ranitidine. C) Glucagon. D) Imipenem. E) None of the above.

102. All of the following genetic mutations are associated with an increased risk of deep venous thrombosis EXCEPT:

A) Factor V Leiden mutation. B) Glycoprotein 1b platelet receptor. C) Heterozygous protein C deficiency. D) Prothrombin 20210G. E) Tissue plasminogen activator.

103. What is the most common side effect of chemotherapy ?

A) Alopecia. B) Diarrhea. C) Febrile neutropenia. D) Mucositis. E) Nausea with or without vomiting.

Page 24: DR. FAHMI KHAN:  · 2017-10-18 · 1. A 71-year-old man undergoes a partial colectomy for colon cancer. He develops temperature of 38.6º C on the third post operative day and rigors

24

104. A 48-year-old woman with stage III breast cancer is undergoing chemotherapy with a regimen that includes doxorubicin. She presents 8 days after her last treatment to the emergency department with a fever of 104.1°F (40.1° C). She has chills, rigors, and a headache. Her chest radiograph, urinalysis, and tunneled intravenous catheter site show no obvious evidence of infection. Her white blood cell count upon presentation is 0.5x109L (0% neutrophils, 50% monocytes, 50% lymphocytes). Blood cultures are drawn peripherally and through the catheter. What is the next step in the treatment of this pati ent?

A) Cefepime 2 g IV q8h and vancomycin B) Cefepime 2 g IV q8h, vancomycin, and voriconazole C) Granulocyte-macrophage colony-stimulating factor after subsequent cycles of

chemotherapy only D) Granulocyte-macrophage colony-stimulating factor now and after subsequent cycles of

chemotherapy E) A and C F) A and D

105. A 51-year-old woman has had several syncopal episodes over the past year. Each episode is characterized by sudden but brief loss of consciousness. She has no chest pain. She has no ankle edema. On brain MRI there is a 1.5 cm cystic area in the left parietal cortex. A chest X-ray shows no cardiac enlargement, and her lung fields are normal. Her serum total cholesterol is 6.5 mmol/L. Which of the following cardiac lesions is she most likely to have?

A) Cardiac amyloidosis. B) Ischemic cardiomyopathy. C) Left atrial myxoma. D) Mitral valve prolapse. E) Tuberculous pericarditis.

106. A 58 year-old male is admitted with a blood pressure of 210/120 and episodic runs of ventricular tachycardia. Investigations confirm the presence of a right adrenal pheochromocytoma. Which one of the following would be the most approp riate initial therapy?

A) Amiodarone. B) Atenolol. C) Lignocaine. D) Phenoxybenxamine. E) Propofenone.

107. Each condition listed below is associated with an increased risk of cancer of the esophagus. Which one is most closely linked to adenocarcinoma of the esophagus?

A) Achalasia. B) Smoking. C) Barrett’s esophagus. D) Tylosis. E) Alcoholism.

Page 25: DR. FAHMI KHAN:  · 2017-10-18 · 1. A 71-year-old man undergoes a partial colectomy for colon cancer. He develops temperature of 38.6º C on the third post operative day and rigors

25

108. A previously healthy 54-year-old man presents to the emergency department complaining of chest pain. His ECG shows an acute inferior wall myocardial infarction. His blood pressure is 90/60 mm Hg. On physical examination, he has jugular vein distention and clear lungs. You should treat him immediately with which of the following:

A) Intravenous fluids. B) Norepinephrine. C) Dopamine. D) Nesiritide. E) Nitroprusside.

109. Which of the following viral causes of acute hepati tis is most likely to cause fulminant hepatitis in a pregnant woman?

A) Hepatitis A. B) Hepatitis B. C) Hepatitis C. D) Hepatitis D. E) Hepatitis E.

110. 58-year-old man presents with fatigue. His physical examination is normal except for the presence of splenomegaly. CBC discloses hematocrit, 29%; platelet count, 90x109 L; WBC, 2.7x109 L; and essentially normal red cell morphology (differential 12% monocytes, 12% granulocytes, and 76% lymphocytes). A bone marrow aspirate and biopsy were performed. The aspirate was dry and the biopsy is pending. Based on the available information, the most likely diagnosis in this case is

A) Chronic lymphocytic leukemia (CLL). B) Hairy cell leukemia. C) Chronic myeloid leukemia (CML). D) Myelofibrosis. E) Multiple myeloma.

111. In evaluation for acute kidney injury in a patient who has recently undergone cardiopulmonary bypass during mitral valve replacement. Which of the following findings on urine microscopy is most suggestive of cholesterol emboli as the source of renal failure?

A) Calcium oxalate crystals. B) Eosinophiluria. C) Granular casts. D) Normal sediment. E) White blood cell casts.

112. Hypersensitivity reactions—such as erythema nodosum, erythema multiforme, arthritis, and arthralgias are most frequently associated with which of the following infections?

A) Histoplasmosis. B) Cryptococcosis. C) Aspergillosis. D) Blastomycosis. E) Coccidioidomycosis.

Page 26: DR. FAHMI KHAN:  · 2017-10-18 · 1. A 71-year-old man undergoes a partial colectomy for colon cancer. He develops temperature of 38.6º C on the third post operative day and rigors

26

113. A 35-year-old woman is seen in clinic for evaluation of dyspnea. Which of the following physical findings would fit the diagnosis of idiopa thic pulmonary arterial hypertension?

A) Elevated neck veins, normal S1 and S2, II/VI diastolic blowing murmur heard at the right upper sternal border.

B) Elevated neck veins; singular, loud S2; II/VI systolic murmur left lower sternal border. C) Elevated neck veins; loud, fixed, split S2; III/VI systolic murmur left lower sternal border. D) Elevated neck veins, expiratory splitting of S2, II/VI harsh systolic murmur left upper

sternal border. E) Elevated neck veins, barrel chest, prolonged expiratory phase.

114. A primary tumor of which of these organs is the least likely to metastasize to bone?

A) Breast. B) Colon. C) Kidney. D) Lung. E) Prostate.

115. A 60 year old woman presented with a small right pupil, right ptosis and impaired sweating over the ipsilateral forehead. Sweating on the rest of the face was unaffected. Where is the most likely site of this lesion?

A) Cervical spinal cord B) Common carotid artery C) Hypothalamus D) Internal carotid artery E) Lateral medulla

116. A 35 year old male recently diagnosed with tuberculosis and is started on medication. After a few days he comes to the emergency department a few hours after taking a meal at McDonald’s with severe pain in the big toe. Blood tests show a very high uric acid level. Which of the following drugs most likely caused the se symptoms?

A) Streptomycin. B) Rifampicin. C) Isoniazid. D) Amikacin. E) Pyrazinamide.

117. Which of the following conditions may give a false/ positive sweat test?

A) Congenital adrenal hyperplasia. B) Hyperthyroidism. C) Hyperparathyroidism. D) Obesity. E) Glucose-6-phosphatase deficiency.

118. Renal failure can result from prerenal, renal, and postrenal pathophysiologic processes. An example of a postrenal cause of renal failure is:

A) Pancreatitis. B) Bacterial endocarditis. C) Methysergide use. D) Indomethacin use. E) Methanol poisoning.

Page 27: DR. FAHMI KHAN:  · 2017-10-18 · 1. A 71-year-old man undergoes a partial colectomy for colon cancer. He develops temperature of 38.6º C on the third post operative day and rigors

27

119. Which of the following physical examination finding s suggests severe aortic regurgitation?

A) Corrigan's pulse. B) Pulsus alternans. C) Pulsus bigeminus. D) Pulsus paradoxus. E) Pulsus parvus et tardus.

120. Which of the following is characteristic of acute m yeloid leukemia when compared with acute lymphocytic leukemia?

A) A higher incidence of meningeal involvement. B) The presence of the t (9:2) chromosomal abnormality (Philadelphia chromosome) as a

prognostic factor. C) An association with prior alkylating agent therapy. D) A higher likehood of achieving complete remission. E) High initial WBC count.

Page 28: DR. FAHMI KHAN:  · 2017-10-18 · 1. A 71-year-old man undergoes a partial colectomy for colon cancer. He develops temperature of 38.6º C on the third post operative day and rigors

28

B: For each question below determine which answer i s true or false

121. Membranoprolypferative glomerulonephritis: A) May be associated with hepatitis C infection. B) Type 1 is associated with subepithelial deposits. C) Type 2 is associated with C3 nephritic factor. D) May be associated with crescent formation. E) Normally lead to end stage renal disease over 10 years.

122. Consumption of raw fish or shellfish is associated with infection caused by

A) Clonorchis sinensis B) Ancylostoma duodenale C) Schistosoma japonicum D) Vibrio parahaemolyticus E) Paragonimus westermani

123. In enteric fever

A) Severe cases should receive high dose dexamethasone B) Ciftiaxone is the treatment of choice for adults in many countries. C) Jaundice is a recognized complication D) Myocarditis is a recognized complication E) If perforation of the bowel occurs it should be managed conservatively because surgery

carries too high a mortality.

124. Features of Bartter’s syndrome include: A) Hypokalemic alkalosis B) Aldosterone deficiency C) Elevated plasma-renin activity D) Resistance to the effect of infused angiotensin E) Atrophy of the juxtaglomerular apparatus

125. Diabetes mellitus is a risk factor for the dev elopment of the following conditions: A) Cataract B) Macular edema C) Glaucoma D) Retinal vein occlusion E) Episcleritis

126. Systematic vasculitis A) Is always associated with positive ANCA B) May occur in association with viral infections. C) Is always limited to small and medium arteries. D) May be associated with hypocomplemetemia. E) Is unlikely to involve renal vasculature in the absence of cutaneous lesions.

127. Recognized features of celiac disease include: A) A protein losing enteropathy B) An association with B27 tissue type C) Associated with disaccharide deficiency D) Interstitial hypermobility E) Skin hyperpigmentation

Page 29: DR. FAHMI KHAN:  · 2017-10-18 · 1. A 71-year-old man undergoes a partial colectomy for colon cancer. He develops temperature of 38.6º C on the third post operative day and rigors

29

128. Hepatitis B. A) Babies born to hepatitis B e antigen positive (HBeAg+ve) mothers should be given

active and passive immunization at birth. B) High risk groups such as hospital workers, homosexual men and intravenous drug

abusers should receive vaccine. C) Co-infection with delta virus may occur in intravenous drug abusers but occurs less

commonly in homosexuals. D) Super-infection with delta virus causes a clinical deterioration. E) Is the major underlying cause of hepatocellular carcinoma.

129. Concerning Ptosis:

A) Congenital origin is almost invariably bilateral B) Is associated with mydriasis when due to complete 3rd nerve lesion C) Results from damage to parasympathetic nerves which run with the occulomotor nerve D) Is associated with paralysis of accommodation when due to damage to the occulomotor

nerve E) Is associated with damage to the cervical sympathetic nervous system

130. Which of the following findings support the di agnosis of acute tubular necrosis rather than prerenal azotemia?

A) Urine osmolality of 300 mOsm/kg. B) Urinary sodium concentration of 10 mmol/day C) Renal failure index of 1.5 D) The fractional excretion of sodium (FENa) of 10% E) The centrifuged sediment of urine reveals pigmented, muddy brown, granular casts.

131. The following are associated with venous thromboses:

A) Systemic lupus erythematosus B) Glanzmann’s syndrome C) Adenocarcinoma D) Protein C deficiency E) Defective fibrinolytic mechanism

132. The following renal diseases are linked to the appropriate mode of inheritance:

A) Cystinosis X-linked B) Hartnup's disease autosomal recessive disorder C) Alport's syndrome autosomal dominant disorder D) Fabry's disease autosomal recessive disorder E) Familial hypophosphatemia Y-linked

133. The following disorders are recognized causes of hemoptysis and renal failure:

A) Goodpasture's syndrome B) Cryoglobulinemia C) Addison's disease D) Leptospirosis E) Carbon tetrachloride poisoning

Page 30: DR. FAHMI KHAN:  · 2017-10-18 · 1. A 71-year-old man undergoes a partial colectomy for colon cancer. He develops temperature of 38.6º C on the third post operative day and rigors

30

134. Prolonged bleeding time is a characteristic feature of: A) Hemophilia B) Thrombocytopenic purpura C) von Willebrand's disease D) Henoch-Schonlein purpura E) Christmas disease

135. Concerning the nerves supplying the upper limb:

A) The musculocutaneous nerve supplies the deltoid muscle B) The radial nerve lies posterior to the shaft of the humerus between the medial and lateral

heads of the triceps muscle C) The median nerve supplies branches to the coracobrachialis D) The ulnar nerve does not supply any muscles in the arm E) The median nerve supplies branches to all the short muscles of the thumb

136. Hepatic involvement is common in:

A) Felty's syndrome B) Still's disease C) Systemic sclerosis D) Cystic fibrosis E) Primary amyloidosis

137. The following are true:

A) HIV-positive hemophiliacs rarely get Kaposi's sarcoma B) Bacterial chest infections are more likely in intravenous drug users C) Staphylococcal arthritis is more common in hemophiliacs D) Cryptosporidium diarrhea is more common in homosexually acquired HIV E) In non-Hodgkin's lymphoma, being HIV positive increases the likelihood of extranodal

disease. 138. In sarcoidosis:

A) A negative transvenous myocardial biopsy excludes the diagnosis B) Clinical manifestations of cardiac involvement occur in 20-30% of patients C) Q waves may occur due to sarcoid disease of the coronary arteries D) Left ventricular aneurysm is a well-recognized complication E) Magnetic resonance imaging may enable the diagnosis to be made

139. Adverse drug reactions:

A) Most commonly affect the cardiovascular and respiratory systems B) Are uncommon in patients taking digoxin and diuretics C) Often affect the gastrointestinal tract and skin D) Are particularly likely to occur in females over 60 years old E) Cause up to 3% of admissions to acute medical wards

140. The following are associated with a reactive arthritis:

A) Campylobacter jejuni B) Chlamydia trachomatis C) Helicobacter pylori D) Mycoplasma E) Shigella flexneri

Page 31: DR. FAHMI KHAN:  · 2017-10-18 · 1. A 71-year-old man undergoes a partial colectomy for colon cancer. He develops temperature of 38.6º C on the third post operative day and rigors

31

141. Therapeutic activity of the following substances depends on their biochemical conversion to active metabolites:

A) Diazepam B) Sulphasalazine C) Enalapril D) Captopril E) Cholecalciferol

142. Psittacosis is:

A) Associated with Horder's spots B) Best treated with ampicillin C) Commonly transmitted from man to man D) May be associated with headache E) Caused by a virus

143. The following are useful in the treatment of Crohn's disease:

A) Cyclosporine B) Metronidazole C) Cholestyramine D) Cyclophosphamide E) Total parenternal nutrition

144. In benign monoclonal gammopathy:

A) Up to 5% of bone marrow cells may be plasma cells B) The incidence of myeloma in the 10 years after diagnosis is <2% C) A normochromic normocytic anemia is common D) There is an increased susceptibility to viral infections E) Soft-tissue plasma-cell tumours are occasionally found

145. Hemochromatosis:

A) Is a genetic defect resulting in copper overload in the liver. B) Is a risk factor for the development of hepatoma. C) Has an equal sex incidence but presents earlier in males than females. D) Is treated by avoiding meat products. E) Can cause hypogonadism in the absence of cirrhosis.

146. The following drugs cause jaundice:

A) Methotrexate. B) Flucloxacillin. C) Metronidazole. D) Isoniazid. E) Phenobarbitone.

147. The following is true of giardiasis:

A) Diarrhea abates with avoidance of dairy produce. B) Diarrhea abates with avoidance of gluten. C) Diarrhea requires treatment with metronidazole. D) Diarrhea is usually accompanied by vomiting. E) Diarrhea commonly results in vitamin B12 deficiency.

Page 32: DR. FAHMI KHAN:  · 2017-10-18 · 1. A 71-year-old man undergoes a partial colectomy for colon cancer. He develops temperature of 38.6º C on the third post operative day and rigors

32

148. The following are features of coeliac disease: A) Hypocalcemia. B) Hypercalcemia. C) Normocytic anemia. D) Hypoalbuminemia. E) Positive antiparietal cell antibodies.

149. Gastric hypomotility (gastroparesis): A) Is commonly associated with diabetes mellitus. B) Is a risk factor for gastro-oesophageal reflux disease. C) Is a feature of generalized scleroderma (systemic sclerosis). D) Occasionally responds to erythromycin. E) Is often secondary to duodenal ulcer disease.

150. A physiological tremor is: A) Present at rest. B) Worsened by anxiety. C) Improved by alcohol. D) Improved by beta-blockers. E) Familial.

151. Stimulation of the sympathetic nervous system: A) Slows the heart rate. B) Enhances myocardial contractility. C) Increases conduction through the atrioventricular node. D) Decreases systemic vascular resistance. E) Increases the plasma renin activity.

152. Causes of a small pupil include: A) Carbon monoxide poisoning. B) Ethylene glycol poisoning. C) Holme's Adie pupil. D) Pontine hemorrhage. E) 3rd Nerve Palsy.

153. If a health care worker experiences an accidental e xposure to HIV, the following steps should be taken:

A) Limit further direct patient contact until it is shown that transmission has not occurred. B) The worker should be offered antibody testing, which includes a baseline test at the time

the accident is reported, again in 6 weeks, 12 weeks and at 6 months. C) Inform the employee not to donate blood pending the outcome of the antibody testing. D) Recommend precaution against possible sexual transmission pending the outcome of

the antibody testing. E) Offer immunoglobulin injection to decrease the risk of acquiring HIV infection.

154. Regarding amiodarone therapy and the lung involveme nt, which of the following is/are true?

A) Bilateral basal crepitations are heard on physical examination. B) Old age predispose to this condition. C) Antibodies are etiologically important. D) Lamellar inclusions bodies are typical. E) Lung toxicity is unpredictable during treatment.

Page 33: DR. FAHMI KHAN:  · 2017-10-18 · 1. A 71-year-old man undergoes a partial colectomy for colon cancer. He develops temperature of 38.6º C on the third post operative day and rigors

33

155. With respect to gastric carcinoma, which of th e following statements is true? A) Aspirin use is a risk factor for gastric carcinoma. B) Helicobacter pylori infection is not associated with gastric carcinoma. C) Esophagogastroduodenoscopy has a diagnostic accuracy of 95%. D) Endoscopic ultrasonography is superior to conventional CT scanning for local tumour

staging. E) Early diagnosis of gastric carcinoma results in a 5 year survival rate of 20%.

156. In the nephropathy of systemic lupus erythematosus:

A) The absence of proteinuria excludes glomerular disease. B) Immunofluorescence shows smooth linear deposits of IgG in glomerular basement

membranes. C) Coagulopathy is a major complication in membranous glomerulonephritis. D) The addition of cyclophosphamide to corticosteroids reduces the incidence of chronic

renal failure. E) Nephrotic syndrome is very rare.

157. Troponins in patients with chronic kidney disease ( CKD), which of the following is/are true?

A) They are used in conjunction with symptoms, electrocardiographic (ECG) changes, and cardiac imaging to diagnose acute myocardial infarction (AMI) in patients with CKD

B) In the absence of other clinical features of ischemia, elevated troponins alone in patients with CKD may not be reliable and may be falsely positive, leading to unnecessary investigations.

C) In many cases, troponin T is more elevated than troponin I in patients with CKD, even at baseline.

D) Asymptomatically elevated troponin levels are associated with a worse long-term prognosis in patients with CKD.

E) Currently, diagnostic guidelines for MI using troponins are different for patients with and without CKD.

158. Regarding parietal lobe lesion consequence, which o f the following is/are true?

A) Bitemporal hemianopia. B) Homonymous inferior quandrantanopia. C) Perseveration. D) Primitive reflexes. E) Wernicke's (receptive ) aphasia.

159. Indication of surgery in treatment of prostatic val ve infective endocarditis includes :

A) Congestive heart failure refractory to standard medical therapy. B) Fungal infection (except that caused by Histoplasma capsulatum). C) Persistent sepsis after 72 hours of appropriate antibiotic treatment. D) Recurrent septic emboli, especially after 2 weeks of antibiotic treatment. E) Rupture of an aneurysm of the sinus of Valsalva.

160. The most common organism to cause CAPD peritonitis is:

A) Staphylococcus Epidermidis. B) Staphylococcus Aureus. C) Streptococcus species. D) E. Coli. E) Pseudomonas.

Page 34: DR. FAHMI KHAN:  · 2017-10-18 · 1. A 71-year-old man undergoes a partial colectomy for colon cancer. He develops temperature of 38.6º C on the third post operative day and rigors

34

161. Regarding pleural effusion, which of the following usually is/are asymptomatic? A) Lupus pleuritic. B) Postcardiac injury syndrome. C) Pulmonary embolism. D) Malignant pleural effusion. E) Trapped lung.

162. Macroglossia is a possible feature of which of the following conditions?

A) Acromegaly B) Marfan's syndrome C) Hurler's syndrome D) Achondroplasia E) Amyloidosis

163. Bilateral parotid gland enlargement is a symptom of

A) Mikulicz's syndrome. B) Infectious mononucleosis. C) Mumps. D) Brucellosis. E) Sarcoidosis.

164. Symptoms characteristic of an acute exacerbation of ulcerative colitis include: A) The development of anemia. B) The occurrence of vertigo following sulfasalazine therapy. C) Macroscopically detected blood in the feces. D) The development of generalized eruptions. E) An increased erythrocyte sedimentation rate.

165. In the carcinoid syndrome: A) Mitral stenosis is a possible complication. B) It may be diagnosed by measuring the vanillylmandelic acid (VMA) concentration in the

urine. C) The application of methysergide is a possible therapeutic intervention. D) Any associated alcohol intake can induce a facial blushing. E) Chronic dyspnea is a recognized complication.

166. Which of the following statements about Charcot's j oint are correct? A) It is caused by Syringomyelia. B) It is caused by diabetes mellitus. C) It is more frequent in females than in males. D) One of its complications is kyphosis. E) Penicillin therapy is effective if this alteration is caused by syphilis.

167. Recurrent syncopes are possible complications: A) In narcolepsy. B) During the Valsalva maneuver. C) In children suffering from whooping cough. D) Of the Shy-Drager syndrome. E) In Adams-Stokes syncope.

Page 35: DR. FAHMI KHAN:  · 2017-10-18 · 1. A 71-year-old man undergoes a partial colectomy for colon cancer. He develops temperature of 38.6º C on the third post operative day and rigors

35

168. Which of the following symptoms are associated with chronic alcoholism? A) Acoustic hallucinations. B) Amnestic syndromes. C) Severe pruritus. D) Degeneration of the corpus callosum. E) Multiple mononeuritis.

169. Brucellosis is usually associated with: A) Bilateral hilar lymphadenopathy which can be observed on the chest x-ray. B) Jaundice. C) Spontaneous remissions of several weeks to months in length. D) Splenomegaly. E) Marked sweating.

170. Neuropsychiatric symptoms of hepatic insufficiency include: A) A reverse sleep pattern. B) Argyll-Robertson pupils. C) Myelopathy with paraplegia. D) Perseveration signs. E) Diagnostic EEG abnormalities.

171. Plasmodium falciparum: A) Causes more severe disease in pregnancy. B) Is associated with recurrent relapses after initial treatment because of liver hypnozoites. C) It is the only malaria parasite causing greater than 20% parasitemia. D) Infection is typically associated with thrombocytopenia. E) It is the only cause of cerebral malaria.

172. In polycythemia rubra vera:

A) The reticulocyte count is typically elevated. B) The leukocyte alkaline phosphatase activity is low. C) 30% of patients exhibit hyperuricosuria. D) The vitamin B12 binding capacity in the serum is elevated. E) The serum total iron level is typically elevated.

173. Characteristics of Korsakoff ′s syndrome include: A) Polyneuritis which is detected in all cases. B) A clear consciousness. C) Impaired judgment. D) Morphological abnormalities in the hypothalamus. E) Confabulation.

174. The side effects of corticosteroids include: A) A loss of collagen. B) Decreased leukocyte migration. C) Avascular bone necrosis. D) Hypercalcemia. E) Increased vascular permeability.

Page 36: DR. FAHMI KHAN:  · 2017-10-18 · 1. A 71-year-old man undergoes a partial colectomy for colon cancer. He develops temperature of 38.6º C on the third post operative day and rigors

36

175. In which of the following conditions is polydactyli a present? A) The Laurence-Moon-Biedl syndrome. B) Marfan's syndrome. C) Turner's syndrome. D) Fanconi′s congenital aplastic anemia. E) A ventricular septal defect.

176. Paradoxical (reversed) splitting of S2 is detected in A) Left bundle branch block. B) Aortic stenosis. C) Atrial septal defect. D) Wolf Parkinson white syndrome. E) Severe pulmonary insufficiency.

177. The following is/are associated with increased risk of aortic dissection A) Hemochromatosis. B) Marfan syndrome. C) Aortic coarctation. D) Pregnancy. E) Bicuspid aortic valve.

178. In Paget's disease of the bone (osteitis deformans) A) The serum alkaline phosphatase activity is normal unless the patient has had a recent

fracture. B) The serum phosphate concentration is typically low. C) There is a high risk of renal stone formation. D) Adequate therapy includes the administration of a high dose of steroids. E) There is a periosteal thickening.

179. Osteoporosis: A) Causes an elevation of the serum calcium concentration. B) Typically causes elevation of the alkaline phosphatase activity. C) Causes pain in the bones. D) Improves during bed rest. E) The response to calcium substitution therapy is usually positive.

180. In fat embolization: A) Skin hemorrhages forming petechiae are rare. B) Cyanosis is present. C) The occurrence of convulsions indicates a poor prognosis. D) Intravenous alcohol injection is a possible therapeutic intervention. E) Hemoptysis is a common symptom.

Page 37: DR. FAHMI KHAN:  · 2017-10-18 · 1. A 71-year-old man undergoes a partial colectomy for colon cancer. He develops temperature of 38.6º C on the third post operative day and rigors

37

C. Each slide followed by best of five question, Ch oose the best answer

181. This patient presents to the emergency department with dizziness and near syncope and with two syncopal attacks during the past two days.

Choose the best answer to interpret this is ECG

A) RBBB with atrial fibrillation. B) RBBB with sinus bradycardia. C) Sinus bradycardia. D) Pacemaker rhythm. E) Complete heart block.

Page 38: DR. FAHMI KHAN:  · 2017-10-18 · 1. A 71-year-old man undergoes a partial colectomy for colon cancer. He develops temperature of 38.6º C on the third post operative day and rigors

38

182. A 19-year-old man, not known to have any chronic illness presents to the emergency department with bouts of palpitation associated with lightheadedness. (see image).

This condition may be associated with all of the fo llowing EXCEPT A) Ebstein's anomaly of the tricuspid valve B) Mitral valve prolapse C) Hypertrophic cardiomyopathy D) Idiopathic dilated cardiomyopathy E) Patent ductus arteriosus

Page 39: DR. FAHMI KHAN:  · 2017-10-18 · 1. A 71-year-old man undergoes a partial colectomy for colon cancer. He develops temperature of 38.6º C on the third post operative day and rigors

39

183. A 21-year-old female presents to the emergency with pruritic skin rash (located on the palms and feet) and fever shortly after she get sore throat.

This condition can result from all of the following EXCEPT

A) Herpes simplex infection B) Mycoplasma pneumoniae infection C) BCG vaccine D) Corticosteroids E) Hemolytic anemia

Page 40: DR. FAHMI KHAN:  · 2017-10-18 · 1. A 71-year-old man undergoes a partial colectomy for colon cancer. He develops temperature of 38.6º C on the third post operative day and rigors

40

184. A 55-year old female with chronic disease presents to your clinic for follow-up.

X-ray findings in this patient may include all EXCE PT A) Jexta-articular osteosclerosis B) Joint space narrowing C) Marginal erosions D) Carpal ankylosis E) Deformities

Page 41: DR. FAHMI KHAN:  · 2017-10-18 · 1. A 71-year-old man undergoes a partial colectomy for colon cancer. He develops temperature of 38.6º C on the third post operative day and rigors

41

185. This 45- year old male patient is admitted to the ICU, after a diagnostic procedure was carried out on a general medical ward.

Regarding the etiology of this condition all are tr ue EXCEPT A) Commonly caused by spontaneous alveolar rupture B) It can occur during bronchial asthma exacerbation. C) Vigorous coughing is a recognized cause of this condition D) It can occur in marijuana smoking persons E) Pulmonary embolism is a recognized cause of this condition.

Page 42: DR. FAHMI KHAN:  · 2017-10-18 · 1. A 71-year-old man undergoes a partial colectomy for colon cancer. He develops temperature of 38.6º C on the third post operative day and rigors

42

186. A 45-year-old patient is admitted to the hospital for acute pancreatitis. A CT taken a week after disease onset. Patient has no pain and he is afebrile and there is no leucocytosis.

What is the imaging finding?

A) Acute necrotic collection B) Pancreatic abscess C) Pancreatic pseudocyst D) Walled-off necrosis E) Normal pancreas

Page 43: DR. FAHMI KHAN:  · 2017-10-18 · 1. A 71-year-old man undergoes a partial colectomy for colon cancer. He develops temperature of 38.6º C on the third post operative day and rigors

43

187. A 25-year old man, presents with fever and severe headache. No seizures. His medical history is significant for AIDS, which was diagnosed three years earlier. He had not been taking antiretroviral drugs. His last CD4 cell count was 282 per mm3 (0.28 × 109

/L), which was measured three months earlier. On examination, she had a temperature of 102°F (38.9°C) and marked wasting. She had no focal neurologic deficits. Brain CT showed a ring-enhancing lesion in the left cerebral hemisphere.

Based on the patient's history, physical examinatio n, and imaging findings, which one of the following is the most likely diagnosis?

A) Human immunodeficiency virus encephalopathy. B) Primary central nervous system lymphoma. C) Progressive multifocal leukoencephalopathy. D) Toxoplasmic encephalitis. E) Tuberculoma.

Page 44: DR. FAHMI KHAN:  · 2017-10-18 · 1. A 71-year-old man undergoes a partial colectomy for colon cancer. He develops temperature of 38.6º C on the third post operative day and rigors

44

188. A 46-year-old woman presented for her annual well-woman examination and Papanicolaou smear. She had no complaints, except for occasional itchiness associated with skin lesions (see accompanying picture) that were symmetrically present in both axillae.

Which of the following is true about this condition : (choose best answer)

A) It is generally genetically determined. B) The most frequent trigger for this condition is lack of insulin in blood in

association with increase in insulin-like growth factor receptors on keratinocytes and fibroblasts.

C) Associated with polycystic ovaries. D) Associated with secondary hypoadrenalism E) Associated with hyperthyroidism.

Page 45: DR. FAHMI KHAN:  · 2017-10-18 · 1. A 71-year-old man undergoes a partial colectomy for colon cancer. He develops temperature of 38.6º C on the third post operative day and rigors

45

189. A 34-year-old man is found unconscious in the transit hall of the airport with respiratory rate of 7/min and blood pressure of 95/60mmHg. He is resuscitated and put on mechanical ventilator. Abdominal X-ray is done (see image).

Which of the following is most appropriate for mana ging this patient?

A) Measuring blood sugar. B) Measuring cardiac enzymes level. C) Immediate colonoscopy. D) Surgical intervention to remove the cause of his unconsciousness. E) Intravenous specific antidote.

Page 46: DR. FAHMI KHAN:  · 2017-10-18 · 1. A 71-year-old man undergoes a partial colectomy for colon cancer. He develops temperature of 38.6º C on the third post operative day and rigors

46

190. A 78-year-old male with hypertension and diabetes is admitted to the emergency department due to sudden onset of double vision earlier the same day. He denies troubles with weakness, sensory loss, or troubles with attention, concentration, thinking, or memory. On exam, he is unable to adduct the right eye while the left abducting eye shows jerky nystagmus with the quick phase towards the opposite site.

Which of the following is true about the affected s ite? (choose the best answer)

A) It connects 6th cranial nerve nucleus with contralteral 3rd cranial nerve nucleus and paramedian pontine reticular formation.

B) It connects 6th cranial nerve nucleus with contralteral 4th cranial nerve nucleus and adjacent horizontal gaze center.

C) It connects 3rd cranial nerve nucleus with contralteral 4th cranial nerve nucleus and adjacent horizontal gaze center.

D) It connects the vestibular nuclei with the 3rd cranial nucleus and the contralteral 6th cranial nerve nuclei.

E) It enables coordination of both eyes during vertical gaze.

Page 47: DR. FAHMI KHAN:  · 2017-10-18 · 1. A 71-year-old man undergoes a partial colectomy for colon cancer. He develops temperature of 38.6º C on the third post operative day and rigors

47

191. A 24-year old male known to have hereditary hemorrhagic telangiectasia is brought by the family to the emergency department because of a generalized tonic-clonic seizure witnessed by them. On examination he has fever, at 38oC, and his level of consciousness is impaired (Glasgow coma scale 6/15). Examination of his cardiorespiratory and abdominal systems is unremarkable. HIV serology is negative. Brain CT was performed. (See image).

The most probable predisposing factor for this cond ition is A) Pulmonary AV malformation. B) Impaired T cell function. C) Impaired B cell function. D) Low complements. E) Low immunoglobulins.

192. A29-year-old college student female presented with 1-month history of “wavy red bumps “underneath her skin. She had been seen by two other clinics before coming to our clinic. She had not responding to previous therapy with a topical antifungal agent and a course of cefalexin.

Page 48: DR. FAHMI KHAN:  · 2017-10-18 · 1. A 71-year-old man undergoes a partial colectomy for colon cancer. He develops temperature of 38.6º C on the third post operative day and rigors

48

The rash was mildly pruritic but not painful. Examination showed many serpiginous vesicles and bullae on an erythematosus base involving the left medial ankle as well as dorsal left feet which has a large tattoo. She has no significant medical or family history.

What is the likely diagnosis? A) Allergic contact dermatitis B) Cutaneous larva migrans C) Epidermal dermatophytosis. D) Erythema migrans E) Larva currens

Page 49: DR. FAHMI KHAN:  · 2017-10-18 · 1. A 71-year-old man undergoes a partial colectomy for colon cancer. He develops temperature of 38.6º C on the third post operative day and rigors

49

193. A 42-year- old Indian patient, known to have T2DM presents with 2-week fever and abdominal pain. Review of systems reveals nausea, and vomiting for the previous 24 hours but no diarrhea. On examination his temperature is 101° F (38.3° C) and there is diffuse abdominal tenderness, predominantly involving the right upper quadrant. His basic metabolic profile, liver function tests, and amylase and lipase levels are within normal limits; his white blood cell count is 17 x 109L. Results of abdominal radiography were nonspecific. Abdominal CT is done (see image).

What is the likely diagnosis?

A) Hepatoma. B) Cavernous Hemangioma. C) Subcapsular biloma. D) Pyogenic hepatic abscess. E) Solitary hepatic cyst.

Page 50: DR. FAHMI KHAN:  · 2017-10-18 · 1. A 71-year-old man undergoes a partial colectomy for colon cancer. He develops temperature of 38.6º C on the third post operative day and rigors

50

194. A 40 year old man presents with acute monoarthritis of the right great toe. Gout confirmed following joint aspiration and examination of the fluid under polarized light microscopy. His medical history is remarkable for kidney transplantation, hypertension for which he is taking medications.

Which of the following drugs is/are responsible for precipitating this condition? A) Hydrochlorothiazide B) Losartan C) Cyclosporine D) A&C E) B&C

Page 51: DR. FAHMI KHAN:  · 2017-10-18 · 1. A 71-year-old man undergoes a partial colectomy for colon cancer. He develops temperature of 38.6º C on the third post operative day and rigors

51

195. A 40-year-old Indian man presents to the emergency department after he had a syncopal event, which his friend witnessed. Apparently, he had several such episodes in the past, though he never sought medical evaluation for them. ECG is done (See image) and troponin study is normal.

What would the ECG suggest? A) Bifascicular block. B) Brugada syndrome. C) Early repolarization. D) Acute pericarditis. E) Acute myocardial ischemia or infarction.

Page 52: DR. FAHMI KHAN:  · 2017-10-18 · 1. A 71-year-old man undergoes a partial colectomy for colon cancer. He develops temperature of 38.6º C on the third post operative day and rigors

52

196. A 48-year-old woman presents with weakness and hemoptysis, but no fever, cough, or sputum. She has a 60-pack-per-year history of smoking. The chest x-ray (CXR) reveals a lung mass with mediastinal widening. On examination, there is a blue-purple discoloration on the upper eyelids associated with edema. She has proximal muscle weakness rated 4+/5, normal reflexes, and sensation.

Which of the following is the most likely diagnosis for her muscle weakness? A) SLE B) Scleroderma C) Dermatomyositis D) Polyarteritis E) Weber-Christian disease

Page 53: DR. FAHMI KHAN:  · 2017-10-18 · 1. A 71-year-old man undergoes a partial colectomy for colon cancer. He develops temperature of 38.6º C on the third post operative day and rigors

53

197. A 60-year-old man presents for evaluation of exercise related dyspnea and chronic dry cough. The symptoms have been present and progressive for several months. There have been no associated fevers or chills. The patient reports a previous history of smoking. There is no history of environmental exposure, cardiac disease, connective tissue disease, or exposure to medications or drugs with these adverse effects. A recent evaluation found no infectious or cardiac explanation for these symptoms and physical examination reveals a patient in no distress. Vital signs are normal. The exam is notable for rales at the bases of the lung fields as well as clubbing. Chest X-ray is shown below and the esinophilic count is normal.

What of the following is the appropriate investigat ion in the management of this case?

A) Conventional chest CT B) High resolution chest CT C) Transbronchial biopsy D) Transthoracic biopsy E) PET scan

Page 54: DR. FAHMI KHAN:  · 2017-10-18 · 1. A 71-year-old man undergoes a partial colectomy for colon cancer. He develops temperature of 38.6º C on the third post operative day and rigors

54

198. A 55-year-old alcoholic man is found at night in the street unconscious. On arrival at emergency department; he remains unconscious. His respiratory rate is 15/min. The chest is clear. Blood pressure and pulse were normal. ECG is done (see image).

What would the ECG suggest? A) Wolff-Parkinson-White syndrome. B) Hyperkalemia C) Right bundle branch block. D) Left bundle branch block E) Hypothermia

Page 55: DR. FAHMI KHAN:  · 2017-10-18 · 1. A 71-year-old man undergoes a partial colectomy for colon cancer. He develops temperature of 38.6º C on the third post operative day and rigors

55

199. A 55-year-old man presents to the ED complaining of mild diffuse abdominal pain. He states that he underwent a routine colonoscopy yesterday and was told “everything is fine.” The pain began upon waking up and is associated with some nausea. He denies fever, vomiting, diarrhea, and rectal bleeding. His BP is 143/71 mm Hg, HR is 87 beats per minute, temperature is 98.9°F, and RR is 16 breaths per minute. His abd omen is tense but only mildly tender. You order baseline laboratory tests. His chest radiograph is seen below.

Which of the following is the most likely diagnosis ? A) Ascending cholangitis. B) Ischemic colitis. C) Biliary pancreatitis. D) Diverticulitis. E) Pneumoperitoneum.

Page 56: DR. FAHMI KHAN:  · 2017-10-18 · 1. A 71-year-old man undergoes a partial colectomy for colon cancer. He develops temperature of 38.6º C on the third post operative day and rigors

56

200. A 56-year-old present presents with a history of increasing tiredness. On examination there is pigmentation of his skin creases and buccal mucosa. His blood pressure is 90/50 mmHg. Investigations are as follows: Blood urea 8.4 mmol/l; Na 130 mmol/l; Potassium 6.1 mmol/l Chloride 96 mmol/l; Bicarbonate 23 mmol/l.

Which of the following tests will confirm the diagn osis? A) Random cortisol measurement. B) Short ACTH stimulation test. C) Long ACTH stimulation test. D) 09:00 Plasma ACTH level and cortisol measurement. E) Abdominal x-ray.

Answer key in the next page, best luck

Page 57: DR. FAHMI KHAN:  · 2017-10-18 · 1. A 71-year-old man undergoes a partial colectomy for colon cancer. He develops temperature of 38.6º C on the third post operative day and rigors

57

Paper 3 I: Best answer II: false or true 1. E 2. B 3. B 4. A 5. D 6. D 7. D 8. A 9. C 10. B 11. E 12. C 13. A 14. E 15. A 16. C 17. D 18. B 19. A 20. C 21. B 22. B 23. E 24. D 25. E 26. C 27. A 28. D 29. E 30. E

31. D 32. A 33. A 34. E 35. D 36. E 37. D 38. E 39. C 40. D 41. B 42. D 43. B 44. D 45. A 46. D 47. D 48. C 49. C 50. C 51. E 52. C 53. C 54. D 55. C 56. B 57. C 58. C 59. A 60. B

61. B 62. E 63. A 64. D 65. C 66. D 67. B 68. C 69. E 70. A 71. A 72. D 73. B 74. C 75. B 76. E 77. D 78. C 79. E 80. D 81. E 82. D 83. C 84. C 85. D 86. A 87. B 88. E 89. A 90. E

91. D 92. B 93. B 94. E 95. C 96. C 97. A 98. B 99. C 100. E 101. E 102. B 103. E 104. A 105. C 106. D 107. C 108. A 109. E 110. B 111. B 112. E 113. B 114. B 115. D 116. E 117. E 118. C 119. A 120. C

121. TFTTT 122. TFFTF 123. TTTTF 124. TFTTF 125. TTTTF 126. FTFTF 127. TFTFT 128. TTTTF 129. FTTTT 130. TFTTT 131. TFTTT 132. FFTTF 133. TTFFF 134. FTTTF 135. FTFTF 136. TTFTF 137. TTTFT 138. FFFTT 139. FFTTT 140. TTFFT 141. FTTFT 142. TFFTF 143. TTTTT 144. TFFFF 145. FTTFT 146. FTFTF

147. FFTFF 148. TFFTF 149. TTTTF 150. FTFTF 151. FTTFT 152. FFFTF 153. FTTTF 154. TTFTT 155. FFTTF 156. FFTTF 157. FFFFT 158. FTFFF 159. TTTTT 160. TFFFF 161. FFFFT 162. TFTFT 163. TFTFT 164. TFTFT 165. TFTTT 166. TTFTF 167. FTTTT 168. TTFTT 169. FFTTT 170. TFTTF 171. TFTTT 172. FFTTF

173. FTTFT 174. TTTFF 175. TFTFT 176. TTFTF 177. FTTTT 178. FFTFF 179. FFTFF 180. FTTFT

III. Slides

181. E 182. E 183. E 184. A 185. E

186. A 187. D 188. C 189. D 190. A

191. A 192. B 193. D 194. D 195. B

196. C 197. B 198. E 199. E 200. D